What is the unit rate for the following point?
(7, 1 3/4)

Answers

Answer 1

Answer:

Step-by-step explanation:

7


Related Questions

I don’t understand if anyone can explain?

Answers

Answer:

Part A: 320 ft²

Part B: 2,304 ft²

Step-by-step explanation:

Part A:

Area of one triangular face of the square pyramid = ½*base*height

base = 32 ft

height = 20 ft

Area = ½*32*20

Area = 320 ft²

Part B:

Total surface area of the square pyramid = area of the square base + area of the 4 triangular faces

= s² + 4(area of 1 triangular face)

s = 32 ft

area of 1 triangular face = 320 ft²

Total surface area = 32² + 4(320) = 1,024 + 1,280

= 2,304 ft²

For this line 3x−4y−12=0, which statement is true? 1- The x-intercept is 4, and the y-intercept is 3. 2-The x-intercept is 4, and the y-intercept is -3 3-The x-intercept is 3, and the y-intercept is -4. 4-The x-intercept is 3, and the y-intercept is 4.

Answers

9514 1404 393

Answer:

  2. The x-intercept is 4, and the y-intercept is -3

Step-by-step explanation:

The given equation is in general form. I find it easier to see the intercepts when the equation is written in standard form:

  3x -4y = 12

Setting y=0 and solving for x, we have the x-intercept:

  3x = 12   ⇒   x = 12/3 = 4

Setting x=0 and solving for y, we have the y-intercept:

  -4y = 12   ⇒   y = 12/-4 = -3

The x-intercept is 4; the y-intercept is -3.

bcore: 9/ 120 5/17 answered Question 7 You deposit $4000 in an account earning 7% interest compounded monthly. How much will you have in th account in 10 years? Submit Question​

Answers

Answer:

8038.65

Step-by-step explanation:

[tex]4000(1+\frac{.07}{12})^{12*10}=8038.65[/tex]

If a distribution for a quantitative variable is thought to be nearly symmetric with very little variation,and a box and whisker plot is created for this distribution,which of the following is true?
A) The box will be quite wide but the whisker will be very short.
B) The left and right-hand edges of the box will be approximately equal distance from the median.
C) The whiskers should be about half as long as the box is wide.
D) The upper whisker will be much longer than the lower whisker.

Answers

Answer:

B). The left and right-hand edges of the box will be approximately equal distances from the median.

Step-by-step explanation:

The 'symmetry' si described as a 'satisfying arrangement of a balanced distribution of the elements of the whole' while a 'symmetry group' is characterized as a group whose elements are all the transformations under which a given object remains invariant and whose group operation is function composition.

As per the given conditions, the second statement asserts a true claim regarding the keeping of edges of left, as well as, right-hand side's boxes at equal intervals from the median. This will help in making the arrangement fulfilling while keeping a little scope for the variation. Thus, option B is the correct answer.

Translate and solve: 82% of what number is 369?

Answers

Answer:

82% of what number is 369

82% of 450 is 369

Answer:

number is 450

Step-by-step explanation:

let the number be x

82% of x=369

[tex]\frac{82}{100}[/tex]x = 369

82x=369*100

x=36900/82

x=450

City A is due north of City B. Find the distance between City A ​( north​ latitude) and City B ​( north​ latitude). Assume that the radius of Earth is 3960 miles.


The distance between City A and City B is ___miles.


Can someone help?! Show me step by step how this is done that I can understand it better.

Answers

Answer:  1679

=========================================================

Explanation:

The notation [tex]47^{\circ}4'[/tex] means "47 degrees, 4 minutes". The "minutes" isn't referring to a time value, but instead they are arc minutes. If we divide one degree into 60 equal pieces, then we form 60 arc minute slices. So in a sense, we are using a round analogue clock to help connect the two ideas.

We can convert to purely degrees through using this formula here

[tex]a^{\circ}b' = a + \frac{b}{60}[/tex]

So,

[tex]47^{\circ}4' = 47 + \frac{4}{60} \approx 47.06667^{\circ}[/tex]

and similarly,

[tex]22^{\circ}46' = 22+ \frac{46}{60} \approx 22.76667^{\circ}[/tex]

Now subtract the two results we got

47.06667-22.76667 = 24.3

The angular distance between the two cities is 24.3 degrees. By "angular distance" I basically mean how far you need to rotate your viewing angle when looking from city A to city B. Imagine that you're able to be situated at the center of the earth.

The circumference of the earth is

C = 2*pi*r

C = 2*pi*3960

C = 24,881.4138164311

which is approximate and the units are in miles. We multiply by the fraction 24.3/360 to find the arc distance along the curve that corresponds to the angle 24.3 degrees. This is because we don't want the whole circumference, but just a small fraction of it.

So (24.3/360)*24,881.4138164311 = 1,679.4954326091

This rounds to 1679

The distance between the two cities is about 1679 miles.

9514 1404 393

Answer:

  1679 miles

Step-by-step explanation:

One minute is 1/60 of a degree, so the difference in latitudes is ...

  47 4/60 -22 46/60 = 24 18/60 = 24.3 . . . degrees

The arc length is given by ...

  s = rθ . . . . . . r = radius, θ = angle in radians

180° is π radians, so the surface distance between the two cities is ...

  s = (3960 mi)(24.3×π/180) = 534.6π mi ≈ 1679.495 mi

The distance between City A and City B is about 1679 miles.

_____

Additional comment

As a good approximation, the distance is about 60 nautical miles per degree of latitude. Using that approximation, and converting from nautical miles to statute miles would give a distance of 1678 miles.

Find the 40th term of the arithmetic sequence with a = 150 and a common difference of d =-4.
1.-6
2.6
3.-2
4.2

Answers

Answer:

1. -6

Step-by-step explanation:

a1 = 150

a2 = a1 - 4 = 150 - 4 = 146

a3 = a2 - 4 = a1 - 4 - 4 = 146 - 4 = 150 - 4 - 4 = 142

...

=>

an = a1 - (n-1)×4

=>

a40 = a1 - 39×4 = 150 - 156 = -6

How is the graph below misleading?


different widths of the bars

too large of increments used

y-axis doesn't begin at zero

broken y-axis

Answers

Answer:

A is the right answer

Step-by-step explanation:

the y axis can start at 0 and the y axis isn't broken. and the increments aren't to large so most reasonable will be A

Answer:

different widths of the bars

Step-by-step explanation:

the y-axis does begin at 0, the increments are reasonable, and the y-axis is not broken

the widths of the bars are visibly different, so that answer makes the most sense.

A group of 12 friends bought tickets for an afternoon concert. However, not all of the friends were able to sit together. Tickets for floor seats cost $15 each, and tickets for tier seats cost $70 each. The total cost of the tickets was $345. How many tickets for tier seats did they purchase

Answers

Answer:

3 tier seats tickets

Step-by-step explanation:

Let the friends that bought floor seats be x and let the friends that bought tier seats be y.

Thus;

x + y = 12 - - - (eq 1)

Floor seats tickets cost $15 each, and tier seats tickets cost $70 each. The total cost of the tickets was $345. Thus;

15x + 70y = 345 - - - (eq 2)

Let's make x the subject in eq (1)

x = 12 - y

Putting this for x in eq 2 gives;

15(12 - y) + 70y = 345

180 - 15y + 70y = 345

55y = 345 - 180

55y = 165

y = 165/55

y = 3

Put 3 for y in x = 12 - y to get;

x = 12 - 3

x = 9

Thus, they purchased 9 floor seats tickets and 3 tier seats tickets

given the points (-4,8)and(6,-12)
1 Determine the midpoint of the line segment connecting the points.
2 Determine the distance separating the two points

Answers

Answer:

1.

[tex]midpoint = ( \frac{ - 4 + 6}{2} , \: \frac{ - 12 + 8}{2} ) \\ = (1, \: - 2)[/tex]

2.

[tex]distance = \sqrt{ {( - 4 - 6)}^{2} + {( - 12 - 8)}^{2} } \\ = \sqrt{500} \\ = 22.4 \: units[/tex]

Determine the equation of the circle graphed below.

Answers

Answer:

[tex](x - 6)^2 + (y - 2)^2 = 4[/tex]

Step-by-step explanation:

Required

The equation of the circle

The equation of a circle is:

[tex](x - h)^2 + (y - k)^2 = r^2[/tex]

Where:

[tex]Center = (h,k)[/tex]

[tex]radius \to r[/tex]

From the graph, we have

[tex](h,k) = (6,2)[/tex]

[tex]r = 2[/tex]

So:

[tex](x - h)^2 + (y - k)^2 = r^2[/tex] becomes

[tex](x - 6)^2 + (y - 2)^2 = 2^2[/tex]

[tex](x - 6)^2 + (y - 2)^2 = 4[/tex]

If the first hill is 20 cm high, the second hill is 0 cm high, and the last hill is 0 cm high, how long does it take the 50 g car to reach the egg?

Answers

25min I think don’t quote me

What number must you add to complete the square? x^2+26x=23

Answers

Answer:

169

Step-by-step explanation:

x²+26x=23

=> to complete the square, the equation should be : x²+26x + (26/2)² = 23 + (26/2)²

<=> x² +26x + 13² = 23 + 13²

<=> x² +26x +169 = 23 + 169

<=> (x+13)² = 192

so, (x+13)² is the square form , and 169 must be added to complete the square

x²+26x=23

For 2ab to complete in (a+b)² formula

2ab=26

a=1

2b=26b=13b²=169

So 169 must be added

Bilal's fishing line was on a spool with a radius of 4 cm. Suddenly, a fish pulled on the line, and the spool spun
16 times before Bilal began to reel in the fish.
What is the distance the fish pulled the fishing line?
Round your answer to the nearest cm.

Answers

Answer:

402 cm

Step-by-step explanation:

Given :

Radius = 4 cm

The Circumference of a circle :

C = 2πr

C = 2 * π * 4 = 8π

Spool is spun 16 times :

Circumference * 16

8π * 16

8 * 3.14 * 16

= 401.92

= 402 cm

Answer:

402 cm

Step-by-step explanation:

20 POINTS!!
Suppose that, based on a sample, the 95% confidence interval for the mean of
a population is (23,39). What was the mean of the sample?
A. 31
B. 37
C. 35
D. 33

Answers

Answer: 31

Step-by-step explanation: just took the test

A video game that usually costs $30.65 is marked down 60%. Kelvin determined that the new price of the game would be $18.39. Look at Kelvin's work and find his error. ($30.65)(0.60) = $18.39

*Please give an explanation longer than 1 sentence :,)

Answers

Answer:

Kelvin’s error is that when he got the final result, that was the amount that was marked down. He still needed to find the price after the original price was marked down by that number wich in this case is $18.39. So using one of his steps, 30.65(0.6)=18.39. We can subtract 30.65 (original price) and 18.39 (mark down price). You’d get 11.26 dollars as the final price.

A trapezoid has bases that measure 9 cm and 5 cm. The height of the figure is 4 cm. What is the area of the trapezoid?
?
O 28 cm
O 36 cm
O 45 cm
O 90 cm?

Answers

Answer:

28 square centimeters

Step-by-step explanation:

The area of a trapezoid (formula):

(a + b) ÷ 2 x h

Where a & b are the bases, and h is height.

Use formula with given measurements:

(9 + 5) ÷ 2 x 4 = 28

Area is measured in square centimeters

(centimeters in this case)

Therefore the area if the trapezoid is 28 cm^2

I really hope this helps!

(2x+8)-(x-8) PLS HELP

Answers

16 : 2 = 8 so it’s technically 8
X+16


Explanation:


(2x+8)-(x-8)

You would first distribute the - into the (x-8)
-1*x= -x -8*-1= 8

(2x+8)+(-x+8)

Now you combine like terms

2x-x=x 8+8=16

16+x or x+16

Plot the points (-6,8)(−6,8)left parenthesis, minus, 6, comma, 8, right parenthesis and (0,8)(0,8)left parenthesis, 0, comma, 8, right parenthesis on the coordinate plane below.

Answers

Answer:

hello I cant help you right now try again later

A)Chris and Alex Were Building a sand Castle. They Decided to make it into a cone shape. After some time, the diameter of the the sand shaped cone was 12in and the height was 7in. What is the exact volume of the sand cone?

B) the kids kept adding sand, ten minutes later the volume of the sand cone had increased by 30%. what is the exact volume of the sand cone now?

Answers

Answer:

A.) 84π | B.) 546π/5

Step-by-step explanation:

A.)

V(Cone) = πr²(h/3) where r = radius and h = height.

r = d/2 where d = diameter.

r = 12/2 = 6

V(Cone) = π(6)²(7/3) = π(36)(7/3) = 84π

B.)

84π · 0.3 = 126π/5

84π + 126π/5 = 546π/5  

PLEASE HELP!! WILL MARK BRAINLIEST TO WHOEVER GETS IT RIGHT

Answers

Answer:

perpendicular, negative reciprocal

Step-by-step explanation:

a negative and a positive slope. 1/5 is the opposite of 5

HELP!!!

A spherical baseball has a diameter of 5 inches and weighs 7 grams per cubic inch. What is the closest weight of the baseball rounded to the nearest gram?

Answers

Answer:

69

Step-by-step explanation:

xvusvsuvtuvqYSQY

what is 2.8436 rounded to the nearest thousandth?​

Answers

Answer:

2.844

Step-by-step explanation:

2.8436 is 2.844 when rounded to the nearest thousandth because the ten thousandth digit is 6, which means you round up for 3. Hope this helps!

Answer:

2.84

Step-by-step explanation:

2.8436 is rounded to the nearest thousands we get 2.84

The values for three sets of data are shown below.

Data
Data Set
Values
1
42, 48, 50, 88, 49
2
63, 29, 35, 28, 30
3
2, 5, 3, 8

Without calculating any statistics, Anna knows that data set 3 would have the least mean absolute deviation among the three sets. Which statement explains how she knows?
Sets 1 and 2 contain outliers.
Set 3 has the least mean.
Set 3 contains an outlier.
Sets 1 and 2 have an odd number of values.
Answer:
data set 3 would have the least mean absolute deviation among the three sets since there is less spread of the data and the data values in set-3 lie close to the mean.

Step-by-step explanation:
The mean absolute deviation is a measure of spread of the data.

If the data values in the given data set are widely spread then we obtain a higher mean absolute deviation.
if the data values of a given data set are close to each other i.e there is a less spread of the data and hence the mean absolute deviation will be low as the data values will lie close to the mean.
We are given three data set as:

set- 1 42, 48, 50, 88, 49

set- 2 63, 29, 35, 28, 30

set- 3 2, 5, 3, 8

Hence, we could observe that the data values in set 1 and set 2 are widely spread.

In set-1 the data value 88 is much higher value as compared to other data values.

Similarly in set-2 the data value 63 is again a much higher value as compared to other data values.

Whereas in set-3 the data values are all closely related and there is not much spread in the data.

Answers

Answer:

it seems like you already have this answered?

Answer:

data set 3 would have the least mean absolute deviation among the three sets since there is less spread of the data and the data values in set-3 lie close to the mean.

Step-by-step explanation:

The mean absolute deviation is a  measure of spread of the data.

If the data values in the given data set are widely spread then we obtain a higher mean absolute deviation.

if the data values of a given data set are close to each other i.e there is a less spread of the data and hence the mean absolute deviation will be low as the data values will lie close to the mean.

We are given three data set as:

set- 1             42, 48, 50, 88, 49

set- 2             63, 29, 35, 28, 30

set- 3                2, 5, 3, 8

Hence, we could observe that the data values in set 1 and set 2 are widely spread.

In set-1 the data value 88 is  much higher value as compared to other data values.

Similarly in set-2 the data value 63 is again a much higher value as compared to other data values.

Whereas in set-3 the data values are all closely related and there is not much spread in the data.

someone please help marking brainliest

Answers

Answer: (1, 4)


Explanation: When using the method of elimination, the goal is to eliminate a variable by either adding or subtracting the 2 equations. For this question, you can choose either to eliminate X or Y. I’ll eliminate X as an example:

In order to eliminate a variable, the same variable in both equations must have the same coefficient.

(1) 3x+y=7
(2) 2x+5y=22

Multiply (1) by 2:

(3) 6x+2y=14

Multiply (2) by 3:

(4) 6x+15y=66

Now that X in both equations has the same coefficient of 6, you can subtract the two equations to officially eliminate the variable and solve for Y:

Subtract (4) from (3):

-13y=-52
y=4

Now that you have the value of Y, substitute that into either one of the equations to get X. I’ll use the first equation as an example:

3x+(4)=7
3x=3
x=1

Therefore, the point of intersection is (1, 4).


Hope this helps シ

what is the solution of the equation 3x=12 round your answer to the nearest ten thousandth

Answers

Answer:

2.2916

Step-by-step explanation:

Imagine bannanas flying into your room? I mean I couldn't imagine that either but im just saying

A factory robot found flaws in 6 out of 36 bulbs . Considering this data, how many flawed bulbs would the robot be expected to find in a batch of 84 bulbs?

Answers

Answer:

14

Step-by-step explanation:

that is the procedure above

Find a polar equation of the conic in terms of r with its focus at the pole. Conic Eccentricity Directrix Parabola e = 1 x = −1

Answers

Answer:

Step-by-step explanation:

From the given information:

The Directrix  x = - 1

The eccentricity  e = 1

Since the Directrix x = - 1, it implies that the directrix appears on the left negative side of the pole and is vertical.

Hence, the conic equation in terms of r is:

[tex]\mathsf{r =\dfrac{ep}{1-e \ cos \ \theta}}[/tex]

From the directrix to the pole, the distance p = 1

So;

[tex]\mathsf{r =\dfrac{1*1}{1-1 \ cos \ \theta}}[/tex]

[tex]\mathbf{r =\dfrac{1}{1- \ cos \ \theta}}[/tex]

PLS HELP WILL GIVE BRAINLIAST !!!

Answers

Answer:

32.5 feet long and 20 feet wide

Step-by-step explanation:

To solve for the length first, you have it measured as 26 in long and each 4 in is 5 ft so you can go ahead and divide 26 in by 4 in and that gives you 6.5 in so what we did was figure out how many 4 in were in 26 in and therefore we have a 6.5 and so just to match that to the 5 ft per each 4 in (in this case we have 6.5) so we multiply 6.5*5 and that equals 32.5 ft which is the length of the garden bed.

Next we are going to solve for the width which is 16 in wide in the scale and so again each 4 in is 5 ft so here again we're going to divide 16 in by 4 in and that equals 4 in and so again each 4 in is 5 ft so we're going to go ahead and multiply 4 * 5 and that gets you 20 ft which is the width of the garden bed.

hi I need help on this

Answers

Answer:

144 cm³ or 144 cubic centimeters

Step-by-step explanation:

Volume = Area triangle * height

Area triangle = 1/2 ( 3 x 8) = 1/2( 24) = 12

12 x 12 = 144 cm³

If my answer is incorrect, pls correct me!

If you like my answer and explanation, mark me as brainliest!

-Chetan K

Other Questions
1. Teacher Laida is an elementary grades teacher. She is very active in communityactivities. Every election time she serves as a member of the election committee. then,during one election period her uncle ran for mayor. Laida got very much involved in thecampaign. Is Teacher Laida's action ethical?a. Yes, because she should support her relative.b. Yes, because she should be non-partisan.C. No, because it is her role to serve the community.d. No, because serving during election time is very dangerous to life. Why is a changing climate important to children and young people What is America's true form of government? I need some help guys Johnny Bravo bought Amazon 25 shares at $50. He sold them at $2346 a few years later. How much money did he make on this stock? Why is this verse an example of iambic pentameter? In a sample of 400 students, 60% of them prefer eBooks. A.Find 98% Confidence Interval for the proportion of all students that prefer ebooksb.b. Find the margin of erro (Acte 1) Quand les membres de la bande sont cerns par les policiers Please help me. I feel really confused and I have no clue on what to do Which of the following has caused the greatest increase in global warming ? * A. Burning of fossil fuels B. Increases in atmospheric nitrogen C. People breathing out D. Turning lights off Peter temporarily takes over Thomas job in his absence,what does this move represent? (10 marks) Flowers, a married taxpayer, purchased an annuity for $64,400 that will pay $700 per month over the life of Flowers and Flowers' spouse. At the time of purchase the couple's joint life expectancy was 23 years. Flowers received payment beginning April 1, year 1, amounting to $6,300 in the first year of the annuity contract. How much is includible in Flowers' gross income in the first year Brainliest question!Whats the digit you need to input in the space with the question mark to complete the code for the safe? Heres a hint to save you some stress: The Safe A08 text on the bottom is meaningless, so dont waste time trying to figure out if that will help you get the answer! please help. i have no clue how to do volume of spheres A student wrote a chemical equation as shown.2HS+ 302 HO + SOWhat should the student do to balance the equation? The Peoples Party was founded to Identify the relationship between sampling error and sample size. Take two samples, one is oil and other is water. Mix the two. You observe that? What is the decay factor that corresponds to a product that decreases its value first by 20%, and than decreases by 40% of its value, and finally decreases by 62% of its value?a.0.22c.0.1824b.0.0496d.0.62 You're expecting visitors who will be demanding Mamet access Before they arrive, you can activate a Guest network that has its own ________ and a different password from the one used with the network where you store all your files.